Đến nội dung

Hình ảnh

Bất đẳng thức Schur và kĩ thuật đổi biến theo các đa thức Viete

* * * - - 2 Bình chọn

  • Please log in to reply
Chủ đề này có 5 trả lời

#1
zaizai

zaizai

    Tiến sĩ diễn đàn toán

  • Thành viên
  • 1380 Bài viết

Bất đẳng thức Schur và kĩ thuật đổi biến theo các đa thức Viete


Phần 1: Đa thức đối xứng 3 biến:

Bất đẳng thức đối xứng 3 biến là một dạng rất thú vị và thường xuất hiện trong các kì thi Olympic. Ta có thể định nghĩa về đa thức đối xứng 3 biến như sau:
Đa thức $F(a,b,c)$ với bộ 3 biến thực a,b,c được hiểu là hàm số có dạng:
$F(a,b,c)=\sum_{s=0}^{N}M_s(a,b,c)$
Trong đó:
$M_s(a,b,c)=\sum_{i+j+k=s}T_{ijk}a^ib^jc^k$
Với $i,j,k\in \mathbb{N}$

Ta định nghĩa đa thức đối xứng 3 biến như sau:
Nếu $F(a,b,c)=F'(a',b',c')$
Trong đó $(a',b',c') $là một hoán vị tùy ý của $(a,b,c)$ thì ta gọi $F(a,b,c) $là một đa thức đối xứng.

Phần 2: Các đa thức đối xứng Viete:
Ta định nghĩa các đa thức đối xứng Viete trong lớp các bài toán bao gồm 3 biến số là các đang thức có dạng:

$p=a+b+c,q=ab+bc+ca,r=abc$


Định lí:
Mọi đa thức đối xứng $F(a,b,c)$ đều có thể biểu diễn dưới dạng các đa thức đối xứng Viete.
Đây là 1 định lí đại số rất cơ bản và quan trọng. Phép chứng minh xin ko được nêu ra ở đây. Bạn có thể tham khảo từ rất nhiều trong các tài liệu về Đa thức đại số.

Phần 3, Làm quen với bất đẳng thức Schur:
Có thể rất nhiều bạn đã quá quen thuộc với bất đẳng thức này cũng với phép chứng minh chỉ gồm 2 dòng cho nó. Tuy nhiên trong bài viết này chúng ta sẽ đưa ra một phép chứng minh "lạ mắt" hơn 1 tí. Đôi khi nó cũng là một việc làm rất thú vị:
Định lí schur:
Cho $a,b,c $là các số thực dương. Khi đó với mọi $r\ge 0$thì
$\fbox{a^r(a-b)(a-c)+b^r(b-c)(b-a)+c^r(c-a)(c-b)\ge 0}$
dấu bằng xảy ra khi và chỉ khi $a=b=c$hoặc $a=b$và $c=0 $cùng các hoán vị của nó.
Chứng minh:
Không mất tính tổng quát ta có thể giả sử $a\ge b\ge c$
Ta có :
$a^r(a-b)(a-c)+b^r(b-c)(b-a)+c^r(c-a)(c-b)\ge 0$
$\Leftrightarrow \sum a^r [(a-b)^2+(a-c)^2-(b-c)^2 ]\ge 0\\ \Leftrightarrow \sum (b-c)^2(c^r+b^r- a^r)\ge 0$
Bất đẳng thức đã qui về dạng chính tắc với:
$S_a=c^r+b^r-a^r \\ S_b=a^r+c^r-b^r \\ S_c=a^r+b^r-c^r$
Dễ thấy ta có $S_b, S_c\ge 0$. Ta cần chứng minh $S_c+S_a\ge 0$
Thật vậy ta có $S_c+S_a=2b^r\ge 0$.
Dấu bằng xảy ra khi và chỉ khi $a=b=c$hoặc$ a=b$và $c=0 $cùng các hoán vị của nó.


Phần 4, Làm mạnh bất đẳng thức Schur và ứng dụng của nó:
Trước khi đi vào phần chính của kĩ thuật này, chúng ta hãy cùng tản mạn với bất đẳng thức Schur bằng việc phát biểu và chứng minh trường hợp mạnh hơn của nó trong trường hợp $r=1$.
Như các bạn đã biết với $r=1$thì bất đẳng thức Schur có dạng:
$a^3 +b^3+c^3 + 3abc\ge ab(a+b)+bc(c+b)+ca(c+a)$
Và xuất phát từ ý tưởng làm mạnh bài toán ta sẽ suy nghĩ ngay tới bất đẳng thức
$2(a^2+b^2)\ge (a+b)^2$suy ra $\sqrt{2(a^2+b^2) }\ge a+b$
Bây giờ là bước 2, làm mạnh theo những gì đã định hướng ta chứng minh bất đẳng thức
$a^3+b^3+c^3+3abc\ge ab\sqrt{2(a^2+b^2) }+bc\sqrt{2(c^2+b^2)}+ca\sqrt {2(c^2+a^2) }$
Lời giải:
Bất đẳng thức cần chứng minh tương đương:
$a^3+b^3+c^3+3abc-ab(a+b)-bc(b+c)-ca(a+c) \\ -ab(\sqrt{2(a^2+b^2)}-a-b)-bc(\sqrt{2(b^2+c^2)}-b-c)-ca(\sqrt{2(c^2+a^2)}-c-a) \ge 0$
$\Leftrightarrow \dfrac{(a+b-c)(a-b)^2+(b+c-a)(b-c)^2+(c+a-b)(c-a)^2}{2} \\ -\dfrac{ab(a-b)^2}{\sqrt{2(a^2+b^2)}+a+b }-\dfrac{bc(b-c)^2}{\sqrt{2(b^2+c^2)}+b+c }-\dfrac{ca(c-a)^2}{\sqrt{2(c^2+a^2)}+c+a}\ge 0$
$\Leftrightarrow \dfrac{1}{2}\sum (a-b)^2[(a+b-c)- \dfrac{2ab}{\sqrt{2(a^2+b^2)}+a+b }]\ge 0$
Như vậy bước biến đổi về dạng chính tắc đã thành công. Ta chuyển sang bước đánh giá.
Không mất tính tổng quát giả sử $a\ge b\ge c$
Ta có:
$S_c=(a+b-c)- \dfrac{2ab}{\sqrt{2(a^2+b^2)}+a+b }=\dfrac{(a+b-c)(\sqrt{2(a^2+b^2)}+a+b)-2ab}{\sqrt{2(a^2+b^2)}+a+b}\\ \ge \dfrac{2(a+b-c)(a+b)-2ab}{\sqrt{2(a^2+b^2)}+a+b}\ge 0$
(do $a+b\ge a$và $a+b-c\ge b$)
Tương tự ta dễ dàng chứng minh $S_b\ge 0$và dẫn đến
$S_b+S_c\ge 0 $bây giờ ta cần chứng minh:
$S_a+S_c=2b-\dfrac{2ab}{\sqrt{2(a^2+b^2)}+a+b}-\dfrac{2bc}{\sqrt{2(b^2+c^2)}+b+c }\\ \ge 2b[1-\dfrac{a}{2(a+b) }-\dfrac{c}{2(b+c) }]=b[\dfrac{b}{a+b}+\dfrac{b}{b+c})]\ge 0$
Vậy bất đẳng thức được chứng minh. Đẳng thức xảy ra khi và chỉ khi $a=b=c $hoặc $a=b$và $c=0$cùng các hoán vị của nó.

Phần 5: Kĩ thuật đổi biến theo các đa thức đối xứng Viete
Phần này là mục tiêu và cũng là nội dung chính của bài viết. Tư tưởng của kĩ thuật này chắc hẳn chỉ nhìn vào chúng ta cũng dễ dàng nhận ra nó gồm 2 bước:
+ Đổi biến một bất đẳng thức đối xứng 3 biến về dạng các đa thức Viete.
+ Sử dụng định lí Schur để chứng minh bất đẳng thức sau khi đã đổi biến.
Dễ dàng biểu diễn các trường hợp của bất đẳng thức schur theo 3 biến p,q,r
Với trường hợp lũy thừa là 0 thì:
$a^0(a-b)(a-c)+b^0(b-c)(b-a)+c^0(c-a)(c-b)\ge 0 \\ \Leftrightarrow pq-9r\ge 0$
Với trường hợp lũy thừa là 1 thì:
$a^1(a-b)(a-c)+b^1(b-c)(b-a)+c^1(c-a)(c-b)\ge 0 \\ \Leftrightarrow p^3-4qp+9r\ge 0$
Với trường hợp lũy thừa là 2 thì:
$a^2(a-b)(a-c)+b^2(b-c)(b-a)+c^2(c-a)(c-b)\ge 0 \\ \Leftrightarrow p^4-5p^2q+4q^2+6pr \ge 0$
Bất đẳng thức phụ:
$p^2\ge 3q, p^3\ge 27r, q^2\ge 3pr, 2p^3+9r\ge 7pq \\ p^2q+3pr\ge 4q^2 \\ p^2q\ge 3pr+2q^2 \\ p^4+3q^2\ge 4p^2a, p^3r+q^3\ge 6pqr \\ ..............$

Còn rất nhiều bất đẳng thức phụ dạng này các bạn hoàn toàn có thể dùng SOS để chứng minh nó.
Để đổi biến 1 bất đẳng thức đối xứng về dạng các đa thức đối xứng Viete ta cần nắm các hằng đẳng thức đáng nhớ sau:
$(a+b)(b+c)(c+a)=pq-r \\ (a+b)(b+c)+(c+a)(a+b)+(c+a)(b+c)=p^2+q \\ a^3+b^3+c^3=p^3-3pq+3r \\ a^2b^2+b^2c^2+c^2a^2=q^2-2pr\\ a^3b^3+b^3c^3+c^3a^3=q^3-3pqr+3r^2 \\ ab(a+b)+bc(b+c)+ca(c+a)=pq-3r \\ a^4+b^4+c^4=p^4-4p^2q+2q^2+4pr \\ab(a^2+b^2)+bc(b^2+c^2)+ca(c^2+a^2)=p^2q-2q^2-pr \\...............$

Chúng ta có thể dễ dàng thiết lập và tìm ra nhiều hằng đẳng thức nữa. Trên đây là những hằng đẳng thức cho các đa thức bậc thấp hơn 4.

Phần 6: Ứng dụng


Bài toán:[ZaiZai]
Cho $a,b,c$ là các số thực không âm thỏa mãn $ab+bc+ca+abc=4$. Chứng minh rằng:
$3(a^2+b^2+c^2)+abc\ge 10$

Ta có bất đẳng thức cần chứng minh tương đương với:
$3p^2-6q+r \ge 10$
$\leftrightarrow 3p^2-7q -6 \ge 0$
Theo bất đẳng thức Schur ta có $ q \le \dfrac{36+p^3}{4p+9}$
$\leftrightarrow 3p^2-7\dfrac{36+p^3}{4p+9}-6 \ge 0$
$\leftrightarrow 5p^3+27p^2-24p-306 \ge 0$
$\leftrightarrow \dfrac{(p-3)(5p^2+42p+102)}{4p+9}\ge 0$
Điều này hiển nhiên đúng do $p\ge 3$ và $5p^2+42p+102\ge 0$
Đẳng thức xảy ra khi và chỉ khi $a=b=c=1$

Bài toán 2:[Balkan Contest]
Cho $a,b,c $ là các số thực dương thỏa mãn $abc=1$
$ 2(a^2+b^2+c^2)+12 \ge 3(a+b+c) + 3(ab+bc+ca)$

Ta viết bất đẳng thức lại dưới dạng:
$2(p^2-2q)+12\ge 3p+3q$

Theo bất đẳng thức schur ta có: $q\le \dfrac{p^3+9}{4p}$
Từ đó ta phải chứng minh:
$2p^2-3p-\dfrac{7(p^3+9)}{4p}+12\ge 0\\ \leftrightarrow \dfrac{(p-3)(p^2-9p+21}{4p}\ge 0$
Điều này hiển nhiên đúng vì $p\ge 3$ (by Am-GM) và $p^2-9p+21=(p-\dfrac{9}{4})^2+\dfrac{3}{4}\ge 0$
Đẳng thức xảy ra khi và chỉ khi $a=b=c=1$

Bài toán 3:[Vasile Cirtoaje]
Cho $a,b,c $ là các số thực dương. Chứng minh rằng
$\dfrac 1{a+b}+\dfrac 1{b+c}+\dfrac 1{c+a}\ge \dfrac{a+b+c}{2(ab+bc+ca)}+\dfrac 3{a+b+c}$
Ta có bất đẳng thức cần chứng minh tương đương với:
$\dfrac{p^{2}+q}{pq-r}\ge \dfrac{p}{2q}+\dfrac{3}{p}$
$\leftrightarrow \dfrac{p^{2}+3}{3p-r}\ge \dfrac{p}{6}+\dfrac{3}{p}$
Từ đó biến đổi biểu thức ta cần phải chứng minh:
$(p^{2}+3)6p-p^{2}(3p-r)-18(3p-r)\ge 0$
$\leftrightarrow 3p^{3}+p^{2}r-36p+18r\ge 0$
Lại có theo bất đẳng thức Schur thì:
$p^{3}-4pq+9r\ge 0 \leftrightarrow p^{3}-12p+9r\ge 0$
Ta có:
$\leftrightarrow 3p^{3}+p^{2}r-36p+18r\ge 0$
$\leftrightarrow3(p^{3}-12p+9r)+r(p^{2}-9)\ge 0$
Mặt khác ta lại có:
$r(p^{2}-9)\ge 0 \leftrightarrow (a-b)^{2}+(b-c)^{2}+(c-a)^{2}\ge 0$
Bài toán được chứng minh.

Bài toán 4:[Phạm Kim Hùng]
Chứng minh rằng nếu $a,b,c$ là các số thực không âm và $k\ge 3$ thì

$\dfrac{1}{a+b}+\dfrac{1}{b+c}+\dfrac{1}{c+a}+\dfrac{k}{a+b+c}\ge \dfrac{2\sqrt{k+1}}{\sqrt{ab+bc+ca}}.$

Lời giải: [tanpham]
Ta cần chứng minh :
$\dfrac{p^2+q}{pq-r}+\dfrac{k}{q} \geq\ \dfrac{2\sqrt{k+1}}{\sqrt{q}}$
Dễ thấy :
$\dfrac{p^2+q}{pq-r}+\dfrac{k}{q} \geq\ \dfrac{p^2+q}{pq}+\dfrac{k}{q}=\dfrac{p}{q}+\dfrac{k+1}{p} \geq\ \dfrac{2\sqrt{k+1}}{\sqrt{q}}$
Đúng theo AM-GM !
Đẳng thức xảy ra khi và chỉ với bộ số:
$(\dfrac{k-1+\sqrt{k^{2}-2k-3}}{2}a,a,0)$ và $(\dfrac{k-1-\sqrt{k^{2}-2k-3}}{2}a,a,0)$

Bài toán 5:[Phạm Kim Hùng]
Chứng minh rằng nếu $a,b,c$ là các số thực không âm thì:
$\dfrac{a}{b+c}+\dfrac{b}{a+c}+\dfrac{c}{b+a}+\dfrac{3 \sqrt[3]{abc}}{2(a+b+c)}\ge 4$.

Sử dụng bất đẳng thức Cauchy-Schawz ta có:
$\dfrac{a}{b+c}+\dfrac{b}{a+c}+\dfrac{c}{b+a}+\dfrac{3 \sqrt[3]{abc}}{2(a+b+c)}$
$\ge \dfrac{(a+b+c)^{2}}{2(ab+bc+ca)}+\dfrac{3 \sqrt[3]{abc}}{2(a+b+c)}$
Chúng ta cần chứng minh:
$\dfrac{(a+b+c)^{2}}{ab+bc+ca}+\dfrac{3 \sqrt[3]{abc}}{a+b+c}\ge 4$
Theo bất đẳng thức schur ta có:$ p^{3}-4pq+9r\ge 0$. Chuẩn hóa $r=abc=1 $khi đó ta có bất đẳng thức tương đương với:
$\dfrac{p^{2}}{q}+\dfrac{3}{p}\ge 4 \leftrightarrow p^{3}+3q\ge 4pq \leftrightarrow (p^{3}-4qp+9r)+(3q-9r)\ge 0$
Điều này hiển nhiên đúng vì:
$p^{3}-4pq+9r\ge 0$ và $3q\ge 9r$
Lại có
$3q=3(ab+bc+ca)\ge 9\sqrt[3]{(abc)^{2}}=9=9r \leftrightarrow 3q\ge 9r$

Bài toán 6:[ZaiZai]
Chứng minh rằng nếu $a,b,c$ là các số thực không âm
$\dfrac{27}{2}\ge 4(ab+bc+ca)+\dfrac{a^2b^2}{a+b}+\dfrac{b^2c^2}{b+c}+\dfrac{c^2a^2}{c+a}$

Có thể dùng SOS hoặc SS sau khi đồng bậc hóa chúng, tuy nhiên có thể giải quyết đơn giản chỉ bằng schur. :( Thử ghi ra phát coi :in
Ta đặt $p=a+b+c,q=ab+bc+ca,r=abc $. Dễ dàng đưa bất đẳng thức cần chứng minh về dạng các biểu thức Viet trên:
$\dfrac{27}{2}\ge 4q+\dfrac{q^3-6pr+3r^2}{3q-r}$
Đến đây sử dụng 2 nhận xét theo schur thì:
$ r\le \dfrac{q}{3} $ và $r\ge \dfrac{4q-9}{3}$
Từ đó bất đẳng thức cần chứng minh tương đương với:
$6q(9-3q)(q+2)\ge 0 $
Cái này hiển nhiên đúng do:
$9-3q=(a+b+c)^2-3(ab+bc+ca)=(a-b)^2+(b-c)^2+(c-a)^2$
Đẳng thức xảy ra khi và chỉ khi $a=b=c=1$

Bài toán 7:[Võ Quốc Bá Cẩn]
Cho $a,b,c$ là các số thực không âm thỏa mãn $ab+bc+ca+6abc=9$. Tìm hằng số k tốt nhất sao cho bất đẳng thức sau là đúng:
$a+b+c+kabc\ge k+3$

Dễ dàng dự đoán $k=3 $là hằng số tốt nhất. Như vậy ta cần chứng minh:
$p+3r\ge 6$ hay $2p-q\ge 3$
Nếu $p\ge 6$ thì ta có điều phải chứng minh. Giả sử ngược lại $p\in [3;6]$ . Khi đó ta xét 2 trường hợp:

+ Trường hợp 1: $p^2\ge 4q$ ta có:
$2p-q\ge 2p-\dfrac{p^2}{4}=\dfrac{(p-2)(6-p)}{4}+3\ge 3$

+ Trường hợp 2: $p^2\le 4q$ ta có:
$27=3q+18r\ge 3q^2+2p(4q-p^2)$
Từ đó suy ra:
$2p-q\ge 2p-\dfrac{2p^3+27}{8p+3} \ge 3$
Thật vậy vì điều này tương đương với:
$(p+1)(p-3)(p-6)\le 0$
Điều này hiển nhiên đúng.

Bài toán 8:[Phạm Kim Hùng]
Chứng minh với các số thực không âm $a,b,c$ thì
$(a^4+b^4+c^4)(ab+bc+ca) \ge (a^2+b^2+c^2)(a^2b^2+b^2c^2+c^2a^2).$

Đặt $p=a+b+c,q=ab+bc+ca,r=abc$. Khi đó ta có biểu thức cần chứng minh tương đương với:
$(p^4-4p^2q+2q^2+4pr)q\ge (p^2-2q)(q^2-2pr)$
Không mất tính tổng quát chuẩn hóa $q=ab+bc+ca=1$. Khi đó ta cần chứng minh:
$p^4-4p^2+2+4pr \ge p^2-2p^3r-2+4pr$
Điều này tương đương với:
$(p^4-5p^2+4+6pr)+(2p^3r-6pr)\ge 0$
Khi đó ta có, theo bất đẳng thức Schur bậc 4:
$(p^4-5p^2+4+6pr)\ge 0$
Lại có:
$2p^3r-6pr=2pr(p^2-3)\ge 0$
Điều này hiển nhiên đúng vì ta có:
$p^2-3=(a+b+c)^2-3(ab+bc+ca)=(a-b)^2+(b-c)^2+(c-a)^2\ge 0$

Bài toán 9:[ZaiZai]
Cho $a,b,c$ là các số thực không âm thỏa mãn $abc=1$. Chứng minh rằng:

$\sqrt[3]{(a^3+1)(b^3+1)(c^3+1)}+1\ge \dfrac{1}{a}+\dfrac{1}{b}+\dfrac{1}{c}$


Ta có
$(a^3+1)(b^3+1)(c^3+1)=2+a^3b^3+b^3c^3+a^3c^3+a^3+b^3+c^3$
Sử dụng kết quả
$a^3+b^3+c^3=p^3-3pq+3$
$a^3b^3+b^3c^3+a^3c^3=q^3-3pq+3$
Ta có
$(a^3+1)(b^3+1)(c^3+1)=p^3-3pq+q^3-3pq+8$
BDT đưa về
$p^3-3pq+q^3-3pq+8 \ge (q-1)^3$
$\Leftrightarrow p^3+3q^2+9\ge 6pq+3q$
Xét các trường hợp
Nếu $q \ge p$ thì theo schur có $p^3+9\ge 4pq $
BDT thành $3q^2\ge 3q+2pq$
Do $q\ge p$ nên BDT thành $q^2\ge 3q$ (đúng)
Nếu $p\ge q$ thì BDT thành
$p^3+3q^2+9q+9\ge 6pq+12q$
AM-GM có
$p^3+3q^2+9q\ge 9pq $
cần có $9pq+9\ge 6pq+12q$
$\Leftrightarrow pq+3 \ge 4q$
do $p\ge q$ nên $pq+3 \ge q^2+3\ge 4q$ (Điều này đúng do$ p\ge 3$)


Còn cập nhật tiếp .......... :in
Bài viết lấy từ diễn đàn VIF - www.bdt.ch06k.com
[/COLOR]

Bài viết đã được chỉnh sửa nội dung bởi zaizai: 12-06-2007 - 08:00


#2
zaizai

zaizai

    Tiến sĩ diễn đàn toán

  • Thành viên
  • 1380 Bài viết
Các ví dụ trên khá là đơn giản và dễ hiểu. Sau đây là một số bài toán dành cho việc luyện tập :D
Bài toán 1: [hungkhtn + zaizai]
Cho $a,b,c$ là các số thực thỏa mãn $a+b+c\ge 3$. Chứng minh rằng:
$(a^2+a+1)(b^2+b+1)(c^2+c+1)\ge 9(ab+bc+ca)$
Bài toán 2:
Cho $a,b,c$ là các số thực dương . Chứng minh rằng:
$2(1+abc)+\sqrt{2(a^2+1)(b^2+1)(c^2+1)}\ge (1+a)(1+b)(1+c)$
Bài toán 3:
Cho $a,b,c$ là các số thực dương thỏa mãn $a+b+c=2$. Chứng minh rằng:
$\dfrac{bc}{a^2+1}+\dfrac{ca}{b^2+1}+\dfrac{ab}{c^2+1}\le 1$
Bài toán 4:
Cho $a,b,c$ là các số thực dương. Chứng minh rằng:
$\dfrac{1}{5-2ab}+\dfrac{1}{5-2bc}+\dfrac{1}{5-2ac}\le 1$
Bài toán 5:
Cho $a,b,c$ là các số thực dương thỏa mãn $a^2+b^2+c^2=1$. Chứng minh rằng:
$21+18abc\ge 13(ab+bc+ca)$

Còn rất nhiều bài toán có thể giải theo kĩ thuật này, đôi khi nó là một công cụ khá hiệu quả và đơn giản ... dù đây là một kĩ thuật ko quá mạnh nhưng nó thật sự có những nét độc đáo riêng :D Một lượng lớn bài tập ko thể nêu ra hết được ... hi vọng các bạn sẽ tiếp tục bổ sung các bài toán đặc sắc hơn :D

Bài viết đã được chỉnh sửa nội dung bởi zaizai: 30-06-2007 - 09:52


#3
supermember

supermember

    Đại úy

  • Hiệp sỹ
  • 1646 Bài viết
Bài 1 ngại quá,ko muốn copy cách của Zai :D,cách mình chỉ đúng với a,b,c>0 :D :D
Bài 2 BĐT cần c/m tương đương:$(p-q-r+1)^2 \geq 0 $
Bài 3 biến đổi tương đương đưa về:$r^2+6qr+3q^2+5 \geq q^3+7q (1)$.
+/Nếu $q \leq 1 =>3q^2+5 \geq q^3+7q $=>đpcm
+/Nếu $\dfrac{4}{3} \geq q \geq 1 $.Theo Schur:$9r \geq 8q-8 $.Thế $r=\dfrac{8}{9}(q-1) $ vào (1) ta có đpcm
Bài 4 giả thiết nằm ở đâu :D :D :D
Bài 5 hình như giả thiết là $a^2+b^2+c^2=3 $.Nếu =1 thì quá hiển nhiên :D
Khi bạn là người yêu Toán, hãy chấp nhận rằng bạn sẽ buồn nhiều hơn vui :)

#4
Hắc Diện Ma Thần

Hắc Diện Ma Thần

    Lính mới

  • Thành viên
  • 5 Bài viết
Bài viết trên hay thật.À,anh Zaizai ơi,cho em hỏi cái trang www.bdt.ch06k.com sao em vào không được nhỉ

#5
zaizai

zaizai

    Tiến sĩ diễn đàn toán

  • Thành viên
  • 1380 Bài viết
thực ra thì nó cũng ko có gì đâu em. Anh tin rằng nhiều người hiểu cái này và sử dụng đỉnh hơn anh vài chục lần :( mà đó cũng chưa phải là toàn bộ về kĩ thuật này tuy nhiên giờ ko còn thời gian để viết nữa nên tạm gác lại .... kĩ thuật này vẫn giải quyết được 1 lượng tương đối các bdt 4 biến hoặc nhiều hơn 4 biến.
Trang hiện tại của VIF bị bandwith vì vậy VIF quyết định sẽ sớm chuyển qua host mới với limit band là 2000GB thay vì 1GB như hiện tại :beer trong thời gian sớm nhất (hiện tại đã reg host. còn đợi chỉnh sửa 1 số thứ ). Mong mọi người thông cảm :(

#6
vuthanhtu_hd

vuthanhtu_hd

    Tiến sĩ Diễn Đàn Toán

  • Hiệp sỹ
  • 1189 Bài viết
Đây là file pdf

File gửi kèm


Bài viết đã được chỉnh sửa nội dung bởi vuthanhtu_hd: 22-04-2009 - 18:00

Nếu một ngày bạn cảm thấy buồn và muốn khóc,hãy gọi cho tôi nhé.
Tôi không hứa sẽ làm cho bạn cười nhưng có thể tôi sẽ khóc cùng với bạn.
Nếu một ngày bạn muốn chạy chốn tất cả hãy gọi cho tôi.
Tôi không yêu cầu bạn dừng lại nhưng tôi sẽ chạy cùng với bạn.
Và nếu một ngày nào đó bạn không muốn nghe ai nói nữa,hãy gọi cho tôi nhé.
Tôi sẽ đến bên bạn và chỉ im lặng.
Nhưng nếu một ngày bạn gọi đến tôi mà không thấy tôi hồi âm...
Hãy chạy thật nhanh đến bên tôi vì lúc đó tôi mới là người cần bạn.

______________________
__________________________________
Vu Thanh TuUniversity of Engineering & Technology





1 người đang xem chủ đề

0 thành viên, 1 khách, 0 thành viên ẩn danh